e-Funktion Zerfall

Neue Frage »

Gast Auf diesen Beitrag antworten »
e-Funktion Zerfall
Hi

folgendes Problem:

Aufgabe:

Dringt Licht in Wasser ein verliert es an Intensität. Pro Meter 1/4 des bis dahin verbliebenen Wertes

A)In welcher Wassertiefe beträgt die Lichtintensität weniger als 1% der ursprüunglichen Intensität ???

Hab folgende Funktion aufgestellt:



1. stimmt die ?? pro m nimmt die Intensität ja um 75% ab deshalb k=-0,75
2. wie lös ich Aufgabe a)

Danke schomal !

mfg
Hot
Gast Auf diesen Beitrag antworten »

hm kann es sein das es ln0,75 heißen muss?
Gast Auf diesen Beitrag antworten »

bitte helft mir unglücklich
yeti777 Auf diesen Beitrag antworten »
RE: e-Funktion Zerfall
Hallo Gast,

ich habe da eine Unklarheit. Nimmt die Intensität pro Meter um 1/4 ab, dh. es verbleiben 3/4 oder nimmt die Intensität pro Meter um 3/4 ab und es verbleibt 1/4?

Gruss yeti
Gast Auf diesen Beitrag antworten »
RE: e-Funktion Zerfall
Zitat:
Original von yeti777
Hallo Gast,

ich habe da eine Unklarheit. Nimmt die Intensität pro Meter um 1/4 ab, dh. es verbleiben 3/4 oder nimmt die Intensität pro Meter um 3/4 ab und es verbleibt 1/4?

Gruss yeti


also die Lichtintensität sinkt pro m auf etwa 1/4 des bis dahin verbliebenen Wertes.

d.h. es nimmt um 75 % ab ?? oder?
Poff Auf diesen Beitrag antworten »
RE: e-Funktion Zerfall
warum immer so 'umständlich' ??

.75^x = .01

x = ln(.01)/ln(.75) = 16
.
 
 
yeti777 Auf diesen Beitrag antworten »

OK! Mit deinen Bezeichnungen ist t die Wassertiefe und f(t) die verbleibende Restintensität des Lichtes, ausgedrückt in [%]. Jetzt musst du doch nur noch die Gleichung exp(-0.75*t) <= 1 nach t auflösen. (Komme leider mit dem Formeleditor nicht zurecht)

Gruss yeti
mYthos Auf diesen Beitrag antworten »

d.h. nach 1 m beträgt die Intensität nur noch das 0,25-fache der Intensität des Anfangswertes.

Das Gesetz, nachdem der "Zerfall" vonstatten geht, lautet



wobei f(0) die Anfangsintensität, s der weg in m und k eine (Zerfalls-)Konstante ist. k ist aber weder 0,75 noch 0,25, sondern wird durch Vergleich der Intensitäten (bei s = 1 m) berechnet:



Logarithmieren der Gleichung, k berechnen ...

Gr
mYthos
yeti777 Auf diesen Beitrag antworten »

Hallo mYthos,

du hast natürlich recht. Ein solcher Schnitzer, wie ich ihn eben machte, sollte mir eigentlich nicht mehr passieren, pfui-pfui-pfui, yeti Hammer

Gruss yeti
Gast Auf diesen Beitrag antworten »

Wenn ich davon ausgehe, dass meine Ausgangsintensität = 100 sieht es so aus oder?







stimmt das??
mYthos Auf diesen Beitrag antworten »

JA, das stimmt.

Nun kommt der zweite Teil der Aufgabe: Mit dem eben errechneten k suchst du jene Wassertiefe, bei der die Intensität nur noch 1% beträgt ...
PK Auf diesen Beitrag antworten »

du brauchst doch nur eine Funtion x := 2 ^-2x zu nehmen, dann funzt es

warum macht ihr es euch mit e- Funktionen so schwer?
Gast Auf diesen Beitrag antworten »

Zitat:
Original von mYthos
JA, das stimmt.

Nun kommt der zweite Teil der Aufgabe: Mit dem eben errechneten k suchst du jene Wassertiefe, bei der die Intensität nur noch 1% beträgt ...


muss ich dann einfach f(x) = 0,01 ???

oder wie berechne ich das?
Gast Auf diesen Beitrag antworten »

Zitat:
Original von PK
du brauchst doch nur eine Funtion x := 2 ^-2x zu nehmen, dann funzt es

warum macht ihr es euch mit e- Funktionen so schwer?


hm und warum geht das damit ???
mYthos Auf diesen Beitrag antworten »

@PK

freilich geht's auch anders, vor allem, wie auch Poff es zwar in aller Kürze - jedoch offensichtlich mit falschem Ergebnis - vorgeführt hat. Im Unterricht wird aber - zwecks besseren Verständnisses - das exponentielle Wachstum anhand einer e-Funktion (sie ergibt sich als Lösung der Wachstums- Differentialgleichung) abgehandelt, was im Prinzip dasselbe ist, nur die Basis ist eine andere.

Man kann dann aber wenn man will, wieder zu einer neuen Basis zusammenfassen. Nicht immer geht es so "glatt" wie zufällig hier in diesem Fall, wo ist.

Wenn das k nicht interessiert, braucht es auch nicht berechnet werden.
Wir gehen mit

k = -ln(0,25)

in die Funktion ein:







...

s =


@Gast

Zitat:
Original von Gast

........
muss ich dann einfach f(x) = 0,01 ???


Ja, genau! Also: , wobei k jetzt bekannt ist, s berechnen.
Gast Auf diesen Beitrag antworten »

k = -ln(0,25)

in die Funktion ein:







...

s =[/quote]

hm kann nicht folgen... wie bekommt man das ln da plötzlich weg ??

hab das jetzt so gemacht


mYthos Auf diesen Beitrag antworten »

Ja, so ist es richtig, bereits nach 3,322 m ist's so richtig düster da unten!
Und -
der ln is deswegen "weg", weil gilt:

Aber wie du es jetzt gerechnet hast, ist's ebenso OK und richtig.

Gr
mYthos
Gast Auf diesen Beitrag antworten »

Zitat:
Original von mYthos
Ja, so ist es richtig, bereits nach 3,322 m ist's so richtig düster da unten!
Und -
der ln is deswegen "weg", weil gilt:

Aber wie du es jetzt gerechnet hast, ist's ebenso OK und richtig.

Gr
mYthos


okay danke für die Hilfe Augenzwinkern ) !!!!!!1
Poff Auf diesen Beitrag antworten »

Zitat:
Original von mYthos
@PK

freilich geht's auch anders, vor allem, wie auch Poff es zwar in aller Kürze - jedoch offensichtlich mit falschem Ergebnis - vorgeführt hat. ...



Dringt Licht in Wasser ein verliert es an Intensität. Pro Meter 1/4 des bis dahin verbliebenen Wertes


... und meine 16 Meter sind RICHTIG *gg*
.
etzwane Auf diesen Beitrag antworten »
RE: e-Funktion Zerfall
Zitat:
Original von Gast
Aufgabe:

Dringt Licht in Wasser ein verliert es an Intensität. Pro Meter 1/4 des bis dahin verbliebenen Wertes

A)In welcher Wassertiefe beträgt die Lichtintensität weniger als 1% der ursprüunglichen Intensität ???


Ich nehme die Aufgabe mal total wörtlich und erhalte mit I(t) = Intensität in der Tiefe t mit t in Meter:

I(t+1) - I(t) = - I(t)/4
also
I(t+1) - 3/4*I(t)=0

Zur Lösung dieser Differenzengleichung setze ich mal an: I(t) = a*k^t mit zu bestimmenden Konstanten a und k . Das ergibt mit I(t+1) = a*k^(t+1)

a*k^(t+1) - 3/4*a*k^t = 0, und nach Kürzung durch a und k^t folgt

k = 3/4, somit bisher I(t) = a*0,75^t

a muss hier unbestimmt bleiben, da weitere Aussagen fehlen, es sei daher a=I0 für die Intensität an der Tiefe 0 m.

Somit: I(t) = I0*0,75^t


Jetzt zur Lösung der Aufgabe: 0,01*I0 = I0*0,75^tx,also 0,01=0,75^tx und log(0,01) = tx*log(0,75),

also: tx = 16,01 m ist die Tiefe, in der die Intensität nur noch 1% der ursprünglichen ist.
mYthos Auf diesen Beitrag antworten »

Hallo,

ihr habt offensichtlich nicht aufmerksam genug die nähere Erklärung von Gast, der ja die Frage gestellt hatte, zu der Aufgabe gelesen:

Zitat:
Original von Gast
Zitat:
Original von yeti777
Hallo Gast,

ich habe da eine Unklarheit. Nimmt die Intensität pro Meter um 1/4 ab, dh. es verbleiben 3/4 oder nimmt die Intensität pro Meter um 3/4 ab und es verbleibt 1/4?

Gruss yeti


also die Lichtintensität sinkt pro m auf etwa 1/4 des bis dahin verbliebenen Wertes.

d.h. es nimmt um 75 % ab ?? oder?


Die Lichtintensität sinkt pro m auf etwa 1/4 des bis dahin verbliebenen Wertes, d.h. es verliert meiner Ansicht nach 3/4 seiner Intensität pro Meter.

Es bleibt (mit dieser Angabe) also bei den 3,322 m, alles andere ist nicht richtig, sorry!
Poff Auf diesen Beitrag antworten »

Zitat:
Original von mYthos
Hallo,

ihr habt offensichtlich nicht aufmerksam genug die nähere Erklärung von Gast, der ja die Frage gestellt hatte, zu der Aufgabe gelesen:
...
...
Die Lichtintensität sinkt pro m auf etwa 1/4 des bis dahin verbliebenen Wertes, d.h. es verliert meiner Ansicht nach 3/4 seiner Intensität pro Meter.

Es bleibt (mit dieser Angabe) also bei den 3,322 m, alles andere ist nicht richtig, sorry!




das ist das was GAST erklärt, ABER NICHT DAS was in
der Aufgabe steht und das Deuten eines Aufgabentextes ist mit
ein Teil der Aufgabe selbst,

somit halte ich mich lieber an den Aufgabentext und nicht an
dessen Deutung durch GAST.

Und dieser Aufgabentext sagt das von mir herausgelesene und
NICHT das von GAST 'gewünschte'.
.
mYthos Auf diesen Beitrag antworten »

@Poff

Solange nicht geklärt ist, ob nun das Licht um 1/4 oder auf 1/4 seiner Intensität abgeschwächt wird, haben wir im Zweifelsfalle beide Recht.

GAST muss, wenn er Gewißheit haben will, nochmals genau den Aufgabentext überprüfen.

Ich habe seine Antwort an Yeti als reinen Aufgabentext interpretiert und nicht als seine Deutung oder "das Gewünschte" hinsichtlich der Aufbereitung der Aufgabe.

Gr
mYthos
Neue Frage »
Antworten »



Verwandte Themen

Die Beliebtesten »
Die Größten »
Die Neuesten »